GMAT - SPLessons

GMAT Verbal Section

Home > > Tutorial
SPLessons 5 Steps, 3 Clicks
5 Steps - 3 Clicks

GMAT Verbal Section

shape Introduction

The GMAT Verbal section is devised to measure the ability of the candidate to read and understand written material, to evaluate arguments, and to correct written material to conform to standard written English. The GMAT Verbal Section includes 36 Questions with an allotted time of 65 minutes with the following: Reading Comprehension, Critical Reasoning, and Sentence Correction.

A. Reading Comprehension In this segment you are given a passage (approx. 350 words) on a topic and multiple choice questions in view of the same.
  • You need not have a top to bottom knowledge of the topic. Or maybe you should be able to
  • Understand the fundamental concept of the passage
  • Grasp the key idea and the connection between the different elements involved
Passage 1 Recent years have brought minority-owned businesses in the United States unprecedented opportunities-as well as new and significant risks. Civil rights activists have long argued that one of (5) the principal reasons why Blacks, Hispanics, and other minority groups have difficulty establishing themselves in business is that they lack access to the sizable orders and subcontracts that are gener- ated by large companies. Now Congress, in appar- (10) ent agreement, has required by law that businesses awarded federal contracts of more than $500,000 do their best to find minority subcontractors and record their efforts to do so on forms filed with the government. Indeed, some federal and local agen- (15) cies have gone so far as to set specific percentage goals for apportioning parts of public works con- tracts to minority enterprises. Corporate response appears to have been sub- stantial. According to figures collected in 1977, (20) the total of corporate contracts with minority busi- nesses rose from $77 million in 1972 to $1. billion in 1977. The projected total of corporate contracts with minority businesses for the early 1980’s is estimated to be over 53 billion per year with no (25) letup anticipated in the next decade. Promising as it is for minority businesses, this increased patronage poses dangers for them, too. First, minority firms risk expanding too fast and overextending themselves financially, since most (30) are small concerns and, unlike large businesses, they often need to make substantial investments in new plants, staff, equipment, and the like in order to perform work subcontracted to them. If, there- after, their subcontracts are for some reason (35) reduced, such firms can face potentially crippling fixed expenses. The world of corporate purchasing can be frustrating for small entrepreneurs who get requests for elaborate formal estimates and bids. Both consume valuable time and resources, and a (40) small company’s efforts must soon result in orders, or both the morale and the financial health of the business will suffer. A second risk is that White-owned companies may seek to cash in on the increasing apportion- (45) ments through formation of joint ventures with minority-owned concerns. Of course, in many instances there are legitimate reasons for joint ventures; clearly, White and minority enterprises can team up to acquire business that neither could (50) acquire alone. But civil rights groups and minority business owners have complained to Congress about minorities being set up as “fronts” with White back- ing, rather than being accepted as full partners in legitimate joint ventures. (55) Third, a minority enterprise that secures the business of one large corporate customer often run the danger of becoming--and remaining—dependent. Even in the best of circumstances, fierce compe- tition from larger, more established companies (60) makes it difficult for small concerns to broaden their customer bases: when such firms have nearly guaranteed orders from a single corporate bene- factor, they may truly have to struggle against complacency arising from their current success. Questions: 1. The primary purpose of the passage is to (A) present a commonplace idea and its inaccuracies (B) describe a situation and its potential drawbacks (C) propose a temporary solution to a problem (D) analyze a frequent source of disagreement (E) explore the implications of a finding Answer: B 2. The passage supplies information that would answer which of the following questions? (A) What federal agencies have set percentage goals for the use of minority-owned businesses in public works contracts? (B) To which government agencies must businesses awarded federal contracts report their efforts to find minority subcontractors? (C) How widespread is the use of minority-owned concerns as “fronts” by White backers seeking to obtain subcontracts? (D) How many more minority-owned businesses were there in 1977 than in 1972? (E) What is one set of conditions under which a small business might find itself financially over-extended? Answer: E 3. According to the passage, civil rights activists maintain that one disadvantage under which minority- owned businesses have traditionally had to labor is that they have (A) been especially vulnerable to governmental mismanagement of the economy (B) been denied bank loans at rates comparable to those afforded larger competitors (C) not had sufficient opportunity to secure business created by large corporations (D) not been able to advertise in those media that reach large numbers of potential customers (E) not had adequate representation in the centers of government power Answer: C 4. The passage suggests that the failure of a large business to have its bids for subcontracts result quickly in orders might cause it to (A) experience frustration but not serious financial harm (B) face potentially crippling fixed expenses (C) have to record its efforts on forms filed with the  government (D) increase its spending with minority subcontractors (E) revise its procedure for making bids for federal contracts and subcontracts Answer: A 5. The author implies that a minority-owned concern that does the greater part of its business with one large corporate customer should (A) avoid competition with larger, more established Concerns by not expanding (B) concentrate on securing even more business from that corporation (C) try to expand its customer base to avoid becoming dependent on the corporation (D) pass on some of the work to be done for the corporation to other minority-owned concerns (E) use its influence with the corporation to promote subcontracting with other minority concerns Answer: C 6. It can be inferred from the passage that, compared with the requirements of law, the percentage goals set by “some federal and local agencies ”(lines 14-15) are (A) more popular with large corporations (B) more specific (C) less controversial (D) less expensive to enforce (E) easier to comply with Answer: B 7. Which of the following, if true, would most weaken the author’s assertion that, in the 1970’s, corporate response to federal requirements (lines 18-19) was substantial (A) Corporate contracts with minority-owned businesses totaled $2 billion in 1979. (B) Between 1970 and 1972, corporate contracts with minority-owned businesses declined by 25 percent. (C) The figures collected in 1977 underrepresented the extent of corporate contracts with minority-owned businesses. (D) The estimate of corporate spending with minority-owned businesses in 1980 is approximately $10 million too high. (E) The $1.1 billion represented the same percentage of total corporate spending in 1977 as did $77 million in 1972. Answer: E 8. The author would most likely agree with which of the following statements about corporate response to working with minority subcontractors? (A) Annoyed by the proliferation of “front” organizations, corporations are likely to reduce their efforts to work with minority-owned subcontractors in the near future. (B) Although corporations showed considerable interest in working with minority businesses in the 1970’s, their aversion to government paperwork made them reluctant to pursue many government contracts. (C) The significant response of corporations in the 1970’s is likely to be sustained and conceivably be increased throughout the 1980’s. (D) Although corporations are eager to cooperate with minority-owned businesses, a shortage of capital in the 1970’s made substantial response impossible. (E) The enormous corporate response has all but eliminated the dangers of overexpansion that used to plague small minority-owned businesses. Answer: C Passage 2 Woodrow Wilson was referring to the liberal idea of the economic market when he said that the free enterprise system is the most efficient economic system. Maximum freedom means (5) maximum productiveness; our “openness” is to be the measure of our stability. Fascination with this ideal has made Americans defy the “Old World” categories of settled possessiveness versus unsettling deprivation, the cupidity of retention (10) versus the cupidity of seizure, a “status quo” defended or attacked. The United States, it was believed, had no status quo ante. Our only “sta- tion” was the turning of a stationary wheel, spin- ning faster and faster. We did not base our (15) system on property but opportunity---which meant we based it not on stability but on mobil- ity. The more things changed, that is, the more rapidly the wheel turned, the steadier we would be. The conventional picture of class politics is (20) composed of the Haves, who want a stability to keep what they have, and the Have-Nots, who want a touch of instability and change in which to scramble for the things they have not. But Americans imagined a condition in which spec- (25) ulators, self-makers, runners are always using the new opportunities given by our land. These eco- nomic leaders (front-runners) would thus he mainly agents of change. The nonstarters were considered the ones who wanted stability, a (30) strong referee to give them some position in the race, a regulative hand to calm manic specula- tion; an authority that can call things to a halt, begin things again from compensatorily stag- gered “starting lines.” (35) “Reform” in America has been sterile because it can imagine no change except through the extension of this metaphor of a race, wider inclu- sion of competitors, “a piece of the action,” as it were, for the disenfranchised. There is no (40) attempt to call off the race. Since our only sta- bility is change, America seems not to honor the quiet work that achieves social interdependence and stability. There is, in our legends, no hero- ism of the office clerk, no stable industrial work (45) force of the people who actually make the system work. There is no pride in being an employee (Wilson asked for a return to the time when everyone was an employer). There has been no boasting about our social workers---they are (50) merely signs of the system’s failure, of opportu- nity denied or not taken, of things to be elimi- nated. We have no pride in our growing interdependence, in the fact that our system can serve others, that we are able to help those in (55) need; empty boasts from the past make us ashamed of our present achievements, make us try to forget or deny them, move away from them. There is no honor but in the Wonderland race we must all run, all trying to win, none (60) winning in the end (for there is no end). Questions: 1. The primary purpose of the passage is to (A) criticize the inflexibility of American economic mythology (B) contrast “Old World” and “New World” economic ideologies (C) challenge the integrity of traditional political leaders (D) champion those Americans whom the author deems to be neglected (E) suggest a substitute for the traditional metaphor of a race Answer: A 2. According to the passage, “Old World” values were based on (A) ability (B) property (C) family connections (D) guild hierarchies (E) education Answer: B 3. In the context of the author’s discussion of regulating change, which of the following could be most probably regarded as a “strong referee” (line 30) in the United States? (A) A school principal (B) A political theorist (C) A federal court judge (D) A social worker (E) A government inspector Answer: C 4. The author sets off the word “Reform” (line 35) with quotation marks in order to (A) emphasize its departure from the concept of settled possessiveness (B) show his support for a systematic program of change (C) underscore the flexibility and even amorphousness of United States society. (D) indicate that the term was one of Wilson’s favorites (E) assert that reform in the United States has not been fundamental Answer: E 5. It can be inferred from the passage that the author most probably thinks that giving the disenfranchised “a piece of the action ” (line 38) is (A) a compassionate, if misdirected, legislative measure (B) an example of Americans’ resistance to profound social change (C) an innovative program for genuine social reform (D) a monument to the efforts of industrial reformers (E) a surprisingly “Old World” remedy for social ills Answer: B 6. Which of the following metaphors could the author most appropriately use to ummarize his own assessment of the American economic system (lines 35-60)? (A) A windmill (B) A waterfall (C) A treadmill (D) A gyroscope (E) A bellows Answer: C 7. It can be inferred from the passage that Woodrow Wilson’s ideas about the economic market (A) encouraged those who “make the system work”(lines 45-46) (B) perpetuated traditional legends about America (C) revealed the prejudices of a man born wealthy (D) foreshadowed the stock market crash of 1929 (E) began a tradition of presidential proclamations on economics Answer: B 8. The passage contains information that would answer which of the following questions? Ⅰ.What techniques have industrialists used to manipulate a free market? Ⅱ.In what ways are “ New World” and “ Old World”economic policies similar? Ⅲ. Has economic policy in the United States tended to reward independent action? (A) Ⅰonly (B) Ⅱonly (C) Ⅲ only (D) Ⅰand Ⅱ only (E) Ⅱand Ⅲ only Answer: C 9. Which of the following best expresses the author’s main point? (A) Americans’ pride in their jobs continues to give them stamina today. (B) The absence of a status quo ante has undermined United States economic structure. (C) The free enterprise system has been only a useless concept in the United States (D) The myth of the American free enterprise system is seriously flawed. (E) Fascination with the ideal of “openness” has made Americans a progressive people. Answer: D Passage 3 No very satisfactory account of the mechanism that caused the formation of the ocean basins has yet been given. The traditional view supposes that the upper mantle of the earth behaves as a (5) liquid when it is subjected to small forces for long periods and that differences in temperature under oceans and continents are sufficient to produce convection in the mantle of the earth with rising convection currents under the mid- (10) ocean ridges and sinking currents under the con- tinents. Theoretically, this convection would carry the continental plates along as though they were on a conveyor belt and would provide the forces needed to produce the split that occurs (15) along the ridge. This view may be correct: it has the advantage that the currents are driven by temperature differences that themselves depend on the position of the continents. Such a back- coupling, in which the position of the moving (20) plate has an impact on the forces that move it, could produce complicated and varying motions. On the other hand, the theory is implausible because convection does not normally occur along lines. and it certainly does not occur along (25) lines broken by frequent offsets or changes in direction, as the ridge is. Also it is difficult to see how the theory applies to the plate between the Mid-Atlantic Ridge and the ridge in the Indian Ocean. This plate is growing on both sides, and (30) since there is no intermediate trench, the two ridges must be moving apart. It would be odd if the rising convection currents kept exact pace with them. An alternative theory is that the sink- ing part of the plate, which is denser than the (35) hotter surrounding mantle, pulls the rest of the plate after it. Again it is difficult to see how this applies to the ridge in the South Atlantic, where neither the African nor the American plate has a sinking part. (40) Another possibility is that the sinking plate cools the neighboring mantle and produces con- vection currents that move the plates. This last theory is attractive because it gives some hope of explaining the enclosed seas, such as the Sea of (45) Japan. These seas have a typical oceanic floor, except that the floor is overlaid by several kilo- meters of sediment. Their floors have probably been sinking for long periods. It seems possible that a sinking current of cooled mantle material (50) on the upper side of the plate might be the cause of such deep basins. The enclosed seas are an important feature of the earth’s surface, and seriously require explanation in because, addi- tion to the enclosed seas that are developing at present behind island arcs, there are a number of (55) older ones of possibly similar origin, such as the Gulf of Mexico, the Black Sea, and perhaps the North Sea. Questions: 1. According to the traditional view of the origin of the ocean basins, which of the following is sufficient to move the continental plates? (A) Increases in sedimentation on ocean floors (B) Spreading of ocean trenches (C) Movement of mid-ocean ridges (D) Sinking of ocean basins (E) Differences in temperature under oceans and continents Answer: E 2. It can be inferred from the passage that, of the following, the deepest sediments would be found in the (A) Indian Ocean (B) Black Sea (C) Mid-Atlantic (D) South Atlantic (E) Pacific Answer: B 3. The author refers to a “conveyor belt ” in line 13 in order to (A) illustrate the effects of convection in the mantle (B) show how temperature differences depend on the positions of the continents (C) demonstrate the linear nature of the Mid-Atlantic Ridge (D) describe the complicated motions made possible by back-coupling (E) account for the rising currents under certain mid-ocean ridges Answer: A 4. The author regards the traditional view of the origin of the oceans with (A) slight apprehension (B) absolute indifference (C) indignant anger (D) complete disbelief (E) guarded skepticism Answer: E 5. According to the passage, which of the following are separated by a plate that is growing on both sides? (A) The Pacific Ocean and the Sea of Japan (B) The South Atlantic Ridge and the North Sea Ridge (C) The Gulf of Mexico and the South Atlantic Ridge (D) The Mid-Atlantic Ridge and the Indian Ocean Ridge (E) The Black Sea and the Sea of Japan Answer: D 6. Which of the following, if it could be demonstrated, would most support the traditional view of ocean formation? (A) Convection usually occurs along lines. (B) The upper mantle behaves as a dense solid. (C) Sedimentation occurs at a constant rate. (D) Sinking plates cool the mantle. (E) Island arcs surround enclosed seas. Answer: A 7. According to the passage, the floor of the Black Sea can best be compared to a (A) rapidly moving conveyor belt (B) slowly settling foundation (C) rapidly expanding balloon (D) violently erupting volcano (E) slowly eroding mountain Answer: B 8. Which of the following titles would best describe the content of the passage? (A) A Description of the Oceans of the World (B) Several Theories of Ocean Basin Formation (C) The Traditional View of the Oceans (D) Convection and Ocean Currents (E) Temperature Differences Among the Oceans of the World Answer: B Passage 4 The fossil remains of the first flying vertebrates, the pterosaurs, have intrigued paleontologists for more than two centuries. How such large creatures, which weighed in some cases as much as a piloted hang-glider (5) and had wingspans from 8 to 12 meters, solved the problems of powered flight, and exactly what these creatures were--reptiles or birds-are among the ques- tions scientists have puzzled over. Perhaps the least controversial assertion about the (10) pterosaurs is that they were reptiles. Their skulls, pelvises, and hind feet are reptilian. The anatomy of their wings suggests that they did not evolve into the class of birds. In pterosaurs a greatly elongated fourth finger of each forelimb supported a winglike membrane. (15) The other fingers were short and reptilian, with sharp claws. In birds the second finger is the principal strut of the wing, which consists primarily of feathers. If the pterosaurs walked on all fours, the three short fingers may have been employed for grasping. When a (20) pterosaur walked or remained stationary, the fourth finger, and with it the wing, could only turn upward in an extended inverted V-shape along each side of the animal’s body. The pterosaurs resembled both birds and bats in (25) their overall structure and proportions. This is not sur- prising because the design of any flying vertebrate is subject to aerodynamic constraints. Both the pterosaurs and the birds have hollow bones, a feature that repre- sents a savings in weight. In the birds, however, these (30) bones are reinforced more massively by internal struts. Although scales typically cover reptiles, the pterosaurs probably had hairy coats. T.H. Huxley rea- soned that flying vertebrates must have been warm- blooded because flying implies a high rate of (35) metabolism, which in turn implies a high internal tem- perature. Huxley speculated that a coat of hair would insulate against loss of body heat and might streamline the body to reduce drag in flight. The recent discovery of a pterosaur specimen covered in long, dense, and (40) relatively thick hairlike fossil material was the first clear evidence that his reasoning was correct. Efforts to explain how the pterosaurs became air- borne have led to suggestions that they launched them- selves by jumping from cliffs, by dropping from trees. (45) or even by rising into light winds from the crests of waves. Each hypothesis has its difficulties. The first wrongly assumes that the pterosaurs’ hind feet rese- mbled a bat’s and could serve as hooks by which the animal could hang in preparation for flight. The second (50) hypothesis seems unlikely because large pterosaurs could not have landed in trees without damaging their wings. The third calls for high waves to channel updrafts. The wind that made such waves however, might have been too strong for the pterosaurs to (55) control their flight once airborne. Questions: 1. It can be inferred from the passage that scientists now generally agree that the (A) enormous wingspan of the pterosaurs enabled them to fly great distances (B) structure of the skeleton of the pterosaurs suggests a close evolutionary relationship to bats (C) fossil remains of the pterosaurs reveal how they solved the problem of powered flight (D) pterosaurs were reptiles (E) pterosaurs walked on all fours Answer: D 2. The author views the idea that the pterosaurs became airborne by rising into light winds created by waves as (A) revolutionary (B) unlikely (C) unassailable (D) probable (E) outdated Answer: B 3. According to the passage, the skeleton of a pterosaur can be distinguished from that of a bird by the (A) size of its wingspan (B) presence of hollow spaces in its bones (C) anatomic origin of its wing strut (D) presence of hooklike projections on its hind feet (E) location of the shoulder joint joining the wing to its body Answer: C 4. The ideas attributed to T.H. Huxley in the passage suggest that he would most likely agree with which of the following statements? (A) An animal’s brain size has little bearing on its ability to master complex behaviors. (B) An animal’s appearance is often influenced by environmental requirements and physical capabilities. (C) Animals within a given family group are unlikely to change their appearance dramatically over a period of time. (D) The origin of flight in vertebrates was an accidental development rather than the outcome of specialization or adaptation. (E) The pterosaurs should be classified as birds, not reptiles. Answer: B 5. It can be inferred from the passage that which of the following is characteristic of the pterosaurs? (A) They were unable to fold their wings when not in use. (B) They hung upside down from branches as bats do before flight. (C) They flew in order to capture prey. (D) They were an early stage in the evolution of the birds. (E) They lived primarily in a forestlike habitat. Answer: A 6.Which of the following best describes the organization of the last paragraph of the passage? (A) New evidence is introduced to support a traditional point of view. (B) Three explanations for a phenomenon are presented, and each is disputed by means of specific information. (C) Three hypotheses are outlined, and evidence supporting each is given. (D) Recent discoveries are described, and their implications for future study are projected (E) A summary of the material in the preceding paragraphs is presented, and conclusions are drawn. Answer: B 7. It can be inferred from the passage that some scientists believe that pterosaurs (A) lived near large bodies of water (B) had sharp teeth for tearing food (C) were attacked and eaten by larger reptiles (D) had longer tails than many birds (E) consumed twice their weight daily to maintain their body temperature Answer: A Passage 5 How many really suffer as a result of labor mar- ket problems? This is one of the most critical yet contentious social policy questions. In many ways, our social statistics exaggerate the degree of hard- (5) ship. Unemployment does not have the same dire consequences today as it did in the 1930’s when most of the unemployed were primary breadwin- ners, when income and earnings were usually much closer to the margin of subsistence, and when there (10) were no countervailing social programs for those failing in the labor market. Increasing affluence, the rise of families with more than one wage earner, the growing predominance of secondary earners among the unemployed, and improved social welfare pro- (15) tection have unquestionably mitigated the conse- quences of joblessness. Earnings and income data also overstate the dimensions of hardship. Among the millions with hourly earnings at or below the minimum wage level, the overwhelming majority (20) are from multiple-earner, relatively affluent families. Most of those counted by the poverty statistics are elderly or handicapped or have family responsibilities which keep them out of the labor force, so the poverty statistics are by no means an (25) accurate indicator of labor market pathologies. Yet there are also many ways our social statistics underestimate the degree of labor-market-related hardship. The unemployment counts exclude the millions of fully employed workers whose wages are (30) so low that their families remain in poverty. Low wages and repeated or prolonged unemployment frequently interact to undermine the capacity for self-support. Since the number experiencing jobless- ness at some time during the year is several times (35)the number unemployed in any month, those who suffer as a result of forced idleness can equal or exceed average annual unemployment, even though only a minority of the jobless in any month really suffer. For every person counted in the monthly (40) unemployment tallies, there is another working part-time because of the inability to find full-time work, or else outside the labor force but wanting a job. Finally, income transfers in our country have always focused on the elderly, disabled, and depen- (45)dent, neglecting the needs of the working poor, so that the dramatic expansion of cash and in-kind transfers does not necessarily mean that those fail- ing in the labor market are adequately protected. As a result of such contradictory evidence, it is (50) uncertain whether those suffering seriously as a result of thousands or the tens of millions, and, hence, whether high levels of joblessness can be tol- erated or must be countered by job creation and (55) economic stimulus. There is only one area of agree- ment in this debate---that the existing poverty, employment, and earnings statistics are inadequate for one their primary applications, measuring the consequences of labor market problems. Questions 1. Which of the following is the principal topic of the passage? (A) What causes labor market pathologies that result in suffering (B) Why income measures are imprecise in measuring degrees of poverty (C) Which of the currently used statistical procedures are the best for estimating the incidence of hardship that is due to unemployment (D) Where the areas of agreement are among poverty, employment, and earnings figures (E) How social statistics give an unclear picture of the degree of hardship caused by low wages and insufficient employment opportunities Answer: E 2. The author uses “labor market problems” in lines 1-2 to refer to which of the following? (A) The overall causes of poverty (B) Deficiencies in the training of the work force (C) Trade relationships among producers of goods (D) Shortages of jobs providing adequate income (E) Strikes and inadequate supplies of labor Answer: D 3. The author contrasts the 1930’s with the present in order to show that (A) more people were unemployed in the 1930’s (B) unemployment now has less severe effects (C) social programs are more needed now (D) there now is a greater proportion of elderly and handicapped people among those in poverty (E) poverty has increased since the 1930’s Answer: B 4.Which of the following proposals best responds to the issues raised by the author? (A) Innovative programs using multiple approaches should be set up to reduce the level of unemployment. (B) A compromise should be found between the positions of those who view joblessness as an evil greater than economic control and those who hold the opposite view. (C) New statistical indices should be developed to measure the degree to which unemployment and inadequately paid employment cause suffering. (D) Consideration should be given to the ways in which statistics can act as partial causes of the phenomena that they purport to measure. (E) The labor force should be restructured so that it corresponds to the range of job vacancies. Answer: C 5.The author’s purpose in citing those who are repeatedly unemployed during a twelve-month period is most probably to show that (A) there are several factors that cause the payment of low wages to some members of the labor force (B) unemployment statistics can underestimate the hardship resulting from joblessness (C) recurrent inadequacies in the labor market can exist and can cause hardships for individual workers (D) a majority of those who are jobless at any one time to not suffer severe hardship (E) there are fewer individuals who are without jobs at some time during a year than would be expected on the basis of monthly unemployment figures Answer: B 6. The author states that the mitigating effect of social programs involving income transfers on the income level of low-income people is often not felt by (A) the employed poor (B) dependent children in single-earner families (C) workers who become disabled (D) retired workers (E) full-time workers who become unemployed Answer: A 7. According to the passage, one factor that causes unemployment and earnings figures to overpredict the amount of economic hardship is the (A) recurrence of periods of unemployment for a group of low-wage workers (B) possibility that earnings may be received from more than one job per worker (C) fact that unemployment counts do not include those who work for low wages and remain poor (D) establishment of a system of record-keeping that makes it possible to compile poverty statistics (E) prevalence, among low-wage workers and the unemployed, of members of families in which others are employed Answer: E 8. The conclusion stated in lines 33-39 about the number of people who suffer as a result of forced idleness depends primarily on the point that (A) in times of high unemployment, there are some people who do not remain unemployed for long (B) the capacity for self-support depends on receiving moderate-to-high wages (C) those in forced idleness include, besides the unemployed, both underemployed part-time workers and those not actively seeking work (D) at different times during the year, different people are unemployed (E) many of those who are affected by unemploy-ment are dependents of unemployed workers Answer: D 9. Which of the following, if true, is the best criticism of the author’s argument concerning why poverty statistics cannot properly be used to show the effects of problems in the labor market? (A) A short-term increase in the number of those in poverty can indicate a shortage of jobs because the basic number of those unable to accept employment remains approximately constant. (B) For those who are in poverty as a result of joblessness, there are social programs available that provide a minimum standard of living. (C) Poverty statistics do not consistently agree with earnings statistics, when each is taken as a measure of hardship resulting from unemployment. (D) The elderly and handicapped categories include many who previously were employed in the labor market. (E) Since the labor market is global in nature, poor workers in one country are competing with poor workers in another with respect to the level of wages and the existence of jobs. Answer: A
B. Critical Reasoning There are around 14 critical reasoning questions in the GMAT verbal section. The passage is in the form of an argument with five answer choices. You should be able to
  • Sort out the helpful data from the irrelevant matter
  • Identify the key points that impact, strengthen or weaken the given argument
You have to carefully go through the argument to logically investigate it, read through the alternatives carefully to be able to choose which choice would be the best possible answer.
1. Sue: Commercial flights currently contribute more carbon dioxide to the atmosphere in one year than does the whole of Africa. If we want to reduce global warming we need to restrict the number of flights we take. Dave: Did you know that by taking one inter-continental flight you cause more pollution than you would in twelve months of car travel? Dave’s response to Sues comment serves to A. reinforce Sues contention that flights are a major contributor to increased carbon dioxide levels B. add more weight to her contention that we should reduce the number of flights we take C. mitigate the force of her argument by suggesting that there is an alternative approach D. suggest an alternative that will reduce the effect of pollution E. question whether she really understands the severity of global warming Answer: B 2. It is not unusual to see the ball fall into a black slot on a roulette wheel four times in a row. But for it to fall five or six times in a row into the same color is very unusual. Therefore you can win money by waiting for a run of five of the same color and then betting against that color. If the roulette wheel in question is a fair wheel, which of the following observations or facts, if it were true, would best reveal a fallacy in the logic? A. If there were a reliable way to win at roulette it would be well-known by now. B. It is hard for a player to keep track of what went before for the time required. C. The probability of getting a particular color decreases with the number of times the color has appeared. D. The probability of getting a particular color is always the same no matter what has gone before. E. A person who makes money this way once or twice, will carry on to lose that money after a few more times. Answer: D 3. On the basis of the Big Bang theory scientists predicted levels of Helium-3 in the universe that are ten times greater than the levels actually observed. According to the original model, Helium-3 is produced when low-mass stars burn up hydrogen and become red giants, as well as being produced in the Big Bang itself. Researchers have now produced a new model in which the Helium-3 produced by a red giant is pushed to the stars interior and burnt up. Hence the Big Bang theory is no longer undermined by Helium-3 data. The two portions in bold-face are related to each other in which of the following ways? A. The first highlights an observation that tends to undermine a particular theory. The second is that theory. B. The first is a fact that undermines a theory. The second is context for accepting that theory. C. The first points to an inconsistency in a particular model; the second is the author’s main conclusion. D. The first is a challenge to a classic theory; the second resolves that challenge. E. The first is a position that the author does not accept; the second is the author’s position. Answer: C 4. The committee on sexual discrimination in the workplace has highlighted Supremo Company as a chief offender. Of the twenty senior executives in the firm, only one is a woman. And of the forty junior executives, only five are female. Supremo could best defend itself against the charges by showing that A. male and female executives at the same level have the same qualifications B. they pay the same salary to senior men and senior women C. ten times more men than women apply for jobs with the company D. the work pressures and long hours make jobs with the company unattractive to married women E. all job applicants who were rejected had fewer qualifications than those accepted Answer: E 5. Josh has twenty years of typing experience behind him; therefore, if you are looking for an efficient typist to enter your data into the new system, you need look no further. The speaker assumes that A. Twenty years of practice ensures typing efficiency B. The type of typing required for the new system is identical to what Josh has been doing C. Josh’s job profile is the best that the new employer is going to get D. Josh is an outstandingly fast and accurate typist E. Josh will fit well into the new office Answer: A 6. It has been suggested that long-term prisoners, on release from jail, be given a reasonable state pension to reduce the likelihood of their resorting to crime. Most people instinctively reject the suggestion as they feel it would be like rewarding criminal activity. The supporters of the prisoner’s pension scheme have criticized those who reject this possibility, by claiming that for the critics...... Which of the following is the most logical completion of the sentence above? A. emotion is more important than justice B. punishment for criminals is more important than crime prevention C. crime prevention is not an important issue D. money has too high a value E. the law should not be concerned with what happens after jail Answer: B 7. Recent studies have highlighted the harmful effects of additives in food (colors, preservatives, flavor enhancers etc.). There are no synthetic substances in the foods we produce at Munch on Foods we use only natural ingredients. Hence you can be sure you are safeguarding your family’s health when you buy our products. Which of the following, if true, would most weaken the contention of Munchon Foods? A. Some synthetic substances are not harmful B. Some natural substances found in foods can be harmful C. Food without additives is unlikely to taste good D. Munchon Foods produces only breakfast cereals E. Without preservatives some foods could cause harm Answer: B 8. A fruit known as alma in certain parts of Asia is an excellent source of vitamin C. A small quantity of the fruit grated and added to salads provides almost all the daily requirement of this vitamin. However, the fruit is very sour. A new process designed to remove most of the sour taste will make the fruit acceptable to American tastes. We are therefore starting to grow this fruit for sale in the United States. The argument above assumes all of the following except A. Americans generally won’t eat very sour foods B. The new process does not remove a significant part of the vitamin content C. That a market exists for a new source of vitamin C D. The fruit can be used only in salads E. Apart from being sour there are no other objections to eating this fruit Answer: D 9. Most scientists agree that new lines of interdisciplinary research are the need of the hour. Even government committees on science have stressed the need for more interdisciplinary projects. Yet, of ten proposals for new interdisciplinary projects last year, only one was successfully funded. Some have suggested that this means that as yet researchers are not coming up with sufficiently persuasive projects, or that their proposals are not of high enough quality, or even that the reputations of these researchers is not high enough. However, the real reason probably lies in the way funding is organized. Funding is still allocated according to the old categories and there are no funds specifically for research that overlaps different subject areas. The two parts in bold-face are related to each other in which of the following ways? A. The first is a finding that the author finds unacceptable; the second is the author’s own position B. The first is a finding that the author attempts to account for; the second is a finding that contradicts the author’s main conclusion. C. The first is a fact that the author attempts to account for. The second is data that explicitly supports the author’s main conclusion. D. The first is a position that the author opposes; the second is the author’s main position. E. The first is a situation that the author finds paradoxical; the second is an assumption that the author uses to reinforce the paradox Answer: C 10. Anton: I sold my house on an internet site last year and was happy with the price. I got a speedy sale and the cost of advertising was insignificant. I would advise you to avoid real estate agents. Barbie: It is in the interest of the real estate agent to get me the best price for my property because he gets a commission based on the selling price. Therefore, when selling my house I will certainly use an agent rather than trying to sell the house by word of mouth, or by advertising in newspapers or on the internet. Barbie’s could strengthen her position by pointing out all of the following except A. Houses of comparable value often obtain a lower price when sold on the internet B. Very few houses are sold on the internet at the moment an so a valid comparison is difficult C. The agent’s service includes many add-on benefits in terms of legal fees, surveyor’s reports and advice that are not available on internet sites D. Some buyers pay the agent to find them a cheap house E. The agent’s commission is usually less than the difference between the internet price and the higher price the agent obtains for you Answer: D 11. Early data on seat-belt use showed that seat-belt wearers were less likely to be killed in road accidents. Hence, it was initially believed that wearing a seat-belt increased survival chances in an accident. But what the early analysts had failed to see was that cautious drivers were more likely to wear the belts and were also less likely to cause big accidents, while reckless drivers were more likely to be involved in big accidents and were less likely to wear the belts. Which of the following, if true, could an opponent of the view presented above best cite as a reason for recommending continued use of seat-belts? A. Careful drivers who are involved in accidents caused by reckless drivers, would be more likely to survive if wearing a belt B. All drivers should be required by law to wear a belt C. The ratio of big to small road accidents is very small D. In fatal accidents seat-belt wearers in the front seat are less likely to survive than those wearing seat belts in the back seat E. On average, careful drivers pay lower insurance premiums than do drivers who have been involved in accidents. Answer: A 12. French cuisine is highly regarded all over the world. Yet in Paris there are more American restaurants selling burgers and fries (which many people now class as junk food) than there are in any other European capital city. Obviously the French are very fond of junk food, and are not too proud to eat it. Which of the following, if true, would most weaken the author’s contention? A. There are also a larger number of Lebanese restaurants in Paris than there are in other European capital cities B. French Cordon Bleu cuisine is very expensive C. The number of French tourists eating in New York burger restaurants is very low D. Junk food is actually has high nutritional value when eaten in moderation E. There are an unusually large number of American tourists in Paris who eat at burger joints Answer: E 13. Scientists investigating a rare metabolic disorder hypothesized that obesity was a predisposing factor in the development of the disease. A study of twenty patients found that, on average, the patients were close to the normal weight for their height. Before concluding that obesity is not a predisposing factor, the researchers would find the answer to which of the following questions most useful? A. Are the patients above or below normal height? B. Were any of the patients underweight when the disorder was diagnosed? C. Does weight loss reduce the severity of the symptoms? D. Have the patients always been close to the normal weight for their heights? E. How many of the patients had obese parents? Answer: D 14. In research designed to investigate the possibility of animals developing friendship with other, unrelated, members of their species, a group of 29 chimpanzees were reared together for 15 years. At the end of that time the chimps were presented with two options for obtaining food: press a lever and feed themselves, or press another identical lever and feed themselves, and at the same time deliver food to the chimp next door. (The chimps were able to see each other). The researchers found that the chimps were no more likely to choose the lever that fed a neighbor. The researchers concluded that the chimps had no concept of friendship. However, one critic has suggested that the animals were in an artificial environment from which little can be concluded, and that, at the least, the test ought to have involved the animals being able to touch. What role do the parts in boldface play in the argument above? A. The first is a position that the critic opposes. The second is a position that the critic supports. B. The first is an observation that supports the researcher’s position. The second is an observation that opposes the researcher’s position. C. The first is a finding on which the researchers base their conclusion. The second is a suggestion that might cast doubt on that finding. D. The first is an observation that supports the critic’s conclusion. The second is the critic’s conclusion. E. The first is part of the evidence that the critic disputes. The second is a suggestion that the researchers do not accept. Answer: C 15. Questions 3-4 refer to the following: Jay: We have too many people working on each of our computers in the office. The high frequency of breakdowns is due to too many people handling the same hardware. Ada: We have just as many people working in our office, yet we hardly need any repairs to our systems. Our systems must be more robust than yours. Ada’s argument would be most strengthened by providing data on the A. actual number of people in the two offices B. type of computers that are in both offices C. ratio of computers to users in her office D. number of visits by computer engineers to service the computers in Jay’s office E. number of computers in Jay’s office Answer: C 16. Refer to the extract in the previous question Jay apparently believes that A. he has the best available hardware B. the frequency of breakdowns in his office is above average C. software specifications are not important in his office D. no other office has a similar ratio of computers to users E. he does not need more people working in his office Answer: B 17. A rare disease, malicitis, is being diagnosed with increasing frequency. The number of cases reported this year is more than double the number reported four years ago. The government should now allocate more funds for treatment and prevention of malicitis. All of the following, if true, would weaken the conclusion except A. funds already available for research in malicitis are currently under-utilized B. a new test employed for the first time this year detects malicitis at a considerably earlier stage in the development of the disease C. the number of cases reported this year represents the same fraction of the population as reported in all of the last five years D. a committee of experts reviewed the funding four years ago E. a private foundation has committed sufficient funds to cover treatment and prevention needs as well as research for the next five years Answer: D 18. Thousands of people have tonsillectomies every year and all live normal lives after the operation. We can conclude, from this observation, that the tonsils have no function in the body. The argument would be most weakened by which of the following, if it were true? A. People live normal lives after appendectomies but the appendix is known to be part of the digestive system. B. Another part of the body can take over the function of the tonsils if they are removed. C. The tonsils have been shown to have a vital role to play in the physiology of laboratory rabbits and guinea pigs. D. The human tonsil develops as part of the immune system, a system of vital importance in defense against disease. E. Tonsillectomies are performed only when the tonsils become seriously infected. Answer: B 19. Photography is no longer an art form. Nowadays everyone has access to digital cameras that only need to be pointed at the subject in order to generate a perfect image. The writer of the argument apparently assumes that A. the selection of the subject is not an important artistic factor in photography B. digital cameras will continue to improve in quality C. digital cameras can never go wrong D. photography with all other types of camera is an art form E. art is not perfect Answer: A 20. The enormous distances between stars are not spaces entirely devoid of matter. The interstellar spaces are filled with dust: very low density matter. This miniscule amount of matter, spread over almost infinite distances, acts like a curtain obscuring the stars that lie behind. If it were not for this material we would see no dark patches in the sky at night: the sky would be entirely covered with stars. The two parts in boldface play what roles in the argument above? A. The first is a suggestion that the author wishes to dispute. The second is hypothesis that the author wishes to explain. B. The first is the main point the author wishes to make. The second is a hypothetical result of accepting that point. C. The first is a fact that the author thinks is important in explaining a certain phenomenon. The second is a result that the author would expect if that fact were not true. D. The first is a speculation that the author wishes to justify. The second is a consequence that would result if that speculation is not true. E. The first is an established fact that the author wishes to explain. The second is a consequence of accepting this fact. Answer: C 21. The number of people diagnosed with dengue fever (which is contracted from the bite of an infected mosquito) in North India this year is twice the number diagnosed last year. The authorities have concluded that measures to control the mosquito population have failed in this region. All of the following, if true, would cast doubt on the authorities conclusion except A. more cases are now reported because of increases in administrative efficiency B. a very high proportion of the cases were in people who had recently returned from neighboring countries C. an effective diagnostic test was introduced about nine months ago D. the disease is prevalent only in some industrialized areas which have shown a dramatic increase in population due to migration E. the incidence (number of cases per thousand) of malaria, also contracted from mosquito bites, has increased Answer: E 22. Questions 4-5 refer to the following: In a recent study of responses to visual images, researchers found that women most frequently gave the rating most attractive to images of male faces that were more feminine in contour, and rated more masculine faces, on average, less attractive. The researchers concluded that modern women prefer men who are less obviously masculine in their facial features. The conclusion would be most severely weakened if which of the following were true? A. Facial features are not the criterion that most women use to decide whether a man is attractive. B. The visual images were computer generated composites of photographs and not pictures of actual men. C. The rating scale was a ten point scale with most attractive scoring 1-2 and least attractive scoring 8-10. D. Most popular male actors have the features that the study allocated to the more masculine category. E. The faces with the more masculine features were all significantly older than those with the feminine features. Answer: E 23. Refer to the extract in the previous question Which is the following is an assumption that the researchers apparently made in this study? A. The women who participated in the study were sufficiently representative of modern women in general. B. Male faces are, in general, attractive to women. C. Visual images are important to women. D. It is impossible to predict what features an ideal face would have. E. Women in previous ages would have preferred more masculine men. Answer: A 24. Red is a color which has powerful effects on human beings as well as animals. A group of psychologists carried out an experiment which confirms the subconscious effects of this color on human behavior. They provided selected sports teams at school and college level with either red or blue shorts and recorded the outcome of the games. The teams wearing red won in a disproportionate number of matches. The psychologists suggested that either the teams wearing red subconsciously felt themselves more powerful, or that the non-red teams were subconsciously intimidated by the red color. Which of the following, if true, would most weaken the psychologist’s suggestion? A. Each team wore red in some matches and blue in others. B. The color blue has the subconscious effect of making human beings less competitive. C. The effect was only observed if all the team members wore white shirts. D. Red signifies danger in some cultures whereas it signifies happiness in others. E. In a subsequent study, teams with all-red outfits were more likely to report that they thought they would win no matter what color the opponents wore. Answer: B 25. It is often thought that our own modern age is unique in having a large number of people who live into old age. It has frequently been assumed that plagues, wars, and harsh working conditions killed off most people in previous ages before they could reach old age. However, recent research shows that in 17th century Europe, for example, people over sixty comprised 10 percent of the population. The studies also revealed that although infant mortality remained high until the 20th century in Europe, people who survived to adulthood could expect to live to be old. The portions in boldface play which of the following roles in the argument above? A. The first is a conclusion that the author supports. The second is data that contradicts that conclusion. B. The first is a finding that the author contests. The second is a finding that the author accepts. C. The first is an assumption that the author thinks is invalid. The second is data that validates that assumption. D. The first is a position that the author opposes. The second is a finding that supports the author’s position. E. The first is a position that the author opposes. The second is an assumption which, if valid, negates the author’s view. Answer: D 26. A marriage counselor noted that couples who have occasional violent arguments are less likely to divorce within the next six months than those who have frequent but less violent arguments. He concluded that frequent arguing is a major factor in the causation of severe marital disharmony. The counselor’s conclusion is most weakened by which of the following observations? A. Couples who have already come to the point of divorce argue continuously over small matters. B. People who have recently divorced are more likely to argue violently when they meet. C. Many people in happy marriages have occasional violent arguments. D. Recently divorced people rarely cite frequent arguments as a cause of marital disharmony E. A significant fraction of couples close to divorce do not talk to each other. Answer: A 27. The Dean claimed that, as a result of continued cutbacks in the budget for pure science research, fewer students are choosing a career in physics, and therefore the number of postgraduate students studying physics is likely to decline. Which of the following, if true, casts most doubt on the Dean’s conclusion? A. The number of students majoring in physics at the undergraduate level has been increasing steadily over the years, a trend that is expected to continue. B. The number of students studying chemistry declined even before cutbacks in research funding were noted. C. Most postgraduate students of physics move to careers in computer science and engineering. D. The Dean’s own university has recently increased the number of staff members teaching physics. E. The budget cutbacks are less severe for the pure sciences than for applied sciences. Answer: C 28. A nature conservancy expert found little support for his campaign to protect toads. He suggested that, even thought the campaign highlighted the vital role the toads played in the ecology of the region, people were unenthusiastic about saving toads as these animals are perceived as unpleasant creatures, and people seldom feel passionate about animals with which they have no positive feelings. The expert’s opinion would be most strengthened by which of the following observations? A. Ecological conservation is an increasingly important concern in the region. B. A recent campaign to save bats achieved a measure of success only after a cartoon bat was adopted as the mascot of the local football team. C. Snakes and lizards also need protection in this region as a result of human activity. D. The campaign to protect toads has been in existence for over five years and yet the toad population continues to decline. E. The children in the local schools were found to have a greater aversion to toads than to snakes. Answer: B 29. Many people report that exposure to certain foods such as cheese, red wine, and chocolate, are associated with the onset of migraine headaches. Other people report that exposure to certain smells (especially strong perfumes) seems to trigger a migraine headache, and some note that exposure to bright and flickering lights can be followed by a migraine. It would seem that a person with a tendency to get migraines should try to find which of these situations is associated with the onset of the headache and then avoid this stimulus. All of the following, if true, would indicate potential problems with the recommendation above except A. the time delay between the trigger and the onset of the headache can make it exceptionally difficult to identify the trigger B. the presence of a known trigger doesn’t always cause a migraine C. in many cases an internal hormonal change triggers a migraine D. in a high proportion of cases the patients report multiple triggers for their headaches E. most of the known triggers are common and almost unavoidable features of modern life Answer: B 30. It is strange that in Sentacity there are so many corner shops selling food items. After all there are many supermarkets in the city which sell food at cheaper prices, and many of these supermarkets are open 24-hours. Which of the following, if true, would be of least help in explaining the paradoxical observation? A. The corner shops are selling specialist food items not available in the supermarkets. B. The supermarkets are mostly located on the outskirts of the city and require residents to use cars or public transport to reach them. C. The main business of the local shops is newspaper distribution and food items represent a small part of their turnover. D. The corner shops are mainly family-owned businesses and have been there for much longer than the supermarkets and are perceived as an important feature of the community. E. The corner shops are willing to make home deliveries. Answer: E 31. Samuel is obviously a bad fisherman. During the past season, in which he and the five members of his team spent four months on a boat together off Dutch Harbor, AK, he caught fewer fish than any of his teammates. Which of the following, if true, most weakens the argument above? A) Two seasons ago, Samuel fished on another boat off Dutch Harbor and caught more fish than any other member of that boat. B) Before becoming a fisherman, Samuel piloted a fishing boat whose members regularly caught record numbers of fish. C) While fishing this past season, Samuel fell sick for a week and did not catch any fish during this time. D) Unlike the other fishermen on his boat, at the order of the captain, Samuel fished this past season with experimental bait. E) Amongst the fishing community in Dutch Harbor, Samuel has a reputation for being an especially bad fisherman. Answer: E 32. Samuel is obviously a bad fisherman. During the past season, in which he and the five members of his team spent four months on a boat together off Dutch Harbor, AK, he caught fewer fish than any of his teammates. Which of the following, if true, most weakens the argument above? A) Two seasons ago, Samuel fished on another boat off Dutch Harbor and caught more fish than any other member of that boat. B) Before becoming a fisherman, Samuel piloted a fishing boat whose members regularly caught record numbers of fish. C) While fishing this past season, Samuel fell sick for a week and did not catch any fish during this time. D) Unlike the other fishermen on his boat, at the order of the captain, Samuel fished this past season with experimental bait. E) Amongst the fishing community in Dutch Harbor, Samuel has a reputation for being an especially bad fisherman. Answer: D 33. Virtually all health experts agree that second-hand smoke poses a serious health risk. After the publication of yet another research paper explicating the link between exposure to second-hand smoke and a shorter life span, some members of the State House of Representatives proposed a ban on smoking in most public places in an attempt to promote quality of life and length of lifespan. Which of the following, if true, provides the most support for the actions of the State Representatives? A) The amount of damaging chemicals and fumes released into the air by cigarette smoke is far less than the amount released from automobiles, especially from older models. B) Banning smoking in most public places will not considerably reduce the percent of the population in the state in question that smokes. C) The state whose legislators are proposing the tough smoking legislation has a relatively high percent of its population that smoke. D) Another state that enacted a similar law a decade ago saw a statistically significant drop in lung-cancer rates among non-smokers. E) A nearby state up-wind has the highest number of smokers in the country. Answer: D 34. Net Neutrality stipulates that Internet service providers (ISP) cannot partition their bandwidth such that different types of Internet communications have different maximum bandwidth capacities. For example, an ISP cannot relegate high bandwidth voice-over-IP (VoIP) traffic to a separate tunnel in an attempt to ensure that users of low-bandwidth functions such as plain-text email are not slowed down by the high-bandwidth users. Some individuals support implementing Net Neutrality on the principle that one group (i.e., users of high-bandwidth services) should not be effectively penalized for the actions of another group (i.e., users of slow-bandwidth services, who have a special traffic lane carved out for them, thereby slowing high-bandwidth users). Which of the following, if true, most seriously weakens the argument of the supporters of Net Neutrality mentioned above? A) The jobs of many high-bandwidth users require these individuals to use high-bandwidth services. B) Placing no restrictions on the bandwidth of individuals who use high-bandwidth services would force ISPs to purchase massive amounts of expensive additional bandwidth, disproportionately increasing the price of access for low-bandwidth users. C) A strong and well respected lobbying firm recently revealed it has been hired by large telecommunications firms to oppose Net Neutrality on the grounds that it infringes upon a private company's ability to do business. D) One country that mandated Net Neutrality saw a decrease in satisfaction of Internet users. E) A recent court ruling upheld the principle that technology companies cannot discriminate in whom they serve or how they serve users. Answer: B 35. Eating beets significantly lowers the risk of cancer, according to an article in a nutritional magazine. The article refers to a study that found that people who consumed one or more beets per day were half as likely to be diagnosed with the disease as people who did not. Which of the following, if true, most weakens the argument in the magazine article? A) Another study found that people who consumed one tablespoon of flax seed oil per day were more than four times less likely to be diagnosed with cancer as those who did not. B) Participants in the study reported consuming no vegetables other than beets. C) The study was only conducted in one city. D) In another experiment, cancer patients who ate one or more beets per day were no more likely to recover than those who ate no beets. E) The participants in the study who ate beets were more likely to exercise regularly than those who did not eat beets. Answer: E 36. Wine Company Representative: The corks of red wine bottles pose a threat to the environment because they are treated with chemicals that are especially toxic in landfills. However, the new cork that our company developed, which will be adopted by the entire red wine industry, represents a solution. Since the new cork is natural and not treated with chemicals, when the industry completes its transition to the new cork, there will no longer be any threat to landfills from red wine corks. Which of the following, if true, most weakens the argument above? A) The industry's transition to the new red wine corks will take years, allowing thousands of old corks to pollute landfills. B) Even after the industry's transition to new corks, a large number of wine bottles with old corks will continue to be consumed. C) The new corks take considerably longer to produce. D) Production of the new cork emits more toxic fumes than were emitted in the production of the old cork. E) The new corks are more expensive than the old corks. Answer: B 37. Political Commentator: In order for a democracy to flourish, it is essential that political and journalistic freedom of expression exist. Even if commentators voice ideas that do not support the current government, a society is strengthened by the variety of views expressed. Yet, our government continues to exercise a stranglehold on certain forms of speech. It is essential that the government loosen its control on the media. Despite the potential short-term instability this may cause the country, it will strengthen the long-term health of the country. Which of the following expresses the conclusion of the argument? A) The government is exercising too much control over the media B) The government needs to open the media and release its hold on certain forms of speech C) Democracy requires a strong and free press D) Even ideas not supportive of democracy can strengthen a government E) The future of the media lies in the hands of the government Answer: B 38. Authors writing detective stories frequently include a brilliant detective and an incompetent investigator who embark on separate paths in an attempt to solve a crime. The separate accounts frequently consist of the incompetent investigator becoming distracted by the criminals' well-planned attempts and the competent detective solving the case after a violent confrontation. Many literary analysts believe authors often choose this storyline in an attempt to provide readers additional complexity and challenge in solving the investigation. Which of the following most logically follows from the statements above? A) A well-written detective story consists of an investigation being undertaken by a competent and incompetent investigator. B) Some authors use an incompetent investigator to show the complexities of an investigation. C) Authors never write stories with incompetent investigators who solve a case correctly. D) Authors can use the separate investigative accounts to make predicting the correct outcome of the investigation more difficult. E) Authors write stories with competent and incompetent investigators to show the complexity of real life. Answer: D 39. Years ago, some in the government's intelligence community feared the work of telecommunications researchers at then-emerging private security firms. The government experts concluded that these private firms posed the biggest risk to successful government espionage. As the private security firms began publicly releasing and advertising encryption algorithms and other security products, these government experts saw support for their conclusion when an encryption algorithm that government experts could not break began appearing in countless emails. Which of the following, if true, most weakens the conclusion of the government experts referred to above? A) Shortly before the government experts reached their conclusions, two private security companies each claimed to have developed "the world's strongest email encryption algorithm" B) The private security firms' decision to advertise their products and sell them publicly led to other members of the private sector and academia scrutinizing the encryption algorithms. C) An open-source encryption algorithm, developed by an academic and freely available from popular websites, is recognized by numerous ex-government code breakers as the most unbreakable algorithm ever developed. D) An enemy government recently succeeded in placing a spy within the government espionage operations referred to above. E) To strengthen the reputation of the private security firms, employees of these firms publish information about the strength of their products and the benefits of using them. Answer: C 40. As a result of implementing an experimental farming system that combined aggressive new fertilization, deep irrigation, and speculative pesticides, the yield on crops at a farm in central California grew consistently and considerably over the past six years. However, yields this year unexpectedly plummeted, causing the farm's owners considerable financial difficulties. Which of the following statements, if true, best explains the unexpected drop in yield? A) As a result of a serious and recent economic recession, the market for the products the farm produced shrank in size during the past year. B) Other farms that began using the experimental system at about the same time also reported an unexpected plummet in yields this year. C) The central California region where the farm in question is located experienced a drought 9 years ago, 3 years ago, and this past year. D) A different experimental system, used on a different crop, failed after two years at a farm in Iowa. Scientists later discovered that chemicals involved in this different system slowly poisoned the soil. E) Significant and sustained increases in agricultural productivity eventually lead to an exhaustion of important nutrients in land. Answer: E 41. On a recent expedition to a remote region of northern Canada, scientists uncovered skeletal remains from about 100,000 years ago. Surprisingly, all the skeletal remains, which included many species from differing biological families and spanned about two thousand years, showed evidence of experiencing temperatures in excess of 1000 degrees Fahrenheit (or 538 degrees Celsius). Which of the following, if true, best explains the apparent paradox between the cold environment and the evidence of the bones experiencing hot temperatures? A) Other scientific research released two years before the expedition showed that the remote region of northern Canada underwent considerable warming in the past 100,000 years. B) Chemical changes that naturally occur during the process of decay in only one north Canadian species produce the same evidence of the species' skeletons being exposed to hot temperatures as the expedition scientists found. C) A little over 103,000 years ago, a large fire is known to have occurred in northern Canada. D) Strong evidence exists that as early as 70,000 years ago, Homo sapiens around the world relied heavily on fire to cook animals. E) In the same expedition and in roughly the same layer of excavation, scientists found rudimentary wood cutting and hunting tools used by early humans. Answer: E 42. Most scientists believe that the decay of the ozone layer is a cause of global warming. With a weaker ozone layer, additional wavelengths of light reach the earth. However, the danger posed by ozone decay is not limited to global warming. The decay of the ozone layer, which enables more harmful wavelengths of light to reach the earth's surface, is also believed to cause permanent eye damage in some animals. Which of the following is most strongly supported by the statements above? A) All wavelengths of light from the sun that damage the eyes of animals are blocked by a healthy ozone layer. B) Some animals do not experience damage to their eyes when exposed to unfiltered waves of light. C) Only a handful of animal species live in places where they are exposed to the damaging light waves. D) The rising temperature of the earth poses a severe threat to animals. E) Some wavelengths of light that engender eye damage in certain animals are more likely to hit the earth due to ozone decay. Answer: E 43. Due to significant advances in biotechnology, experts predict that within years, doctors will be able to trace the genetic roots of common medical problems such as depression and bi-polar syndrome. As a result, some physicians predict that these conditions and others like them will be all but eliminated through early identification and genetic therapy. The argument above is based most heavily upon which of the following assumptions? A) There is one and only one strategy for eliminating common medical problems such as depression. B) Common medical problems such as depression are based entirely on treatable genetic malformations. C) Within years, genetics will be the only important scientific field in detecting and treating common medical problems such as depression. D) Every member of the human race has at least one genetic defect. E) Each human defect or sickness can be traced in part to genetics. Answer: B 44. A political party considered by many citizens extreme and incompetent is working hard at overcoming what it perceives as an unfair and prejudiced public image. The group believes it is stereotyped simply because it opposes any preemptive military action, which is unpopular, and calls for a 50% reduction in carbon emissions, a cut far more severe than most citizens want. Which of the following, if true, would most weaken the party's belief about the source of its poor public image? A) The current ruling political party, whose popularity is at near record levels, launched a preemptive military strike against a neighboring country three weeks ago. B) A neighboring country recently suffered a surprise attack that devastated its country and polluted its environment. C) A popular rival party bans unilateral attacks to defend the homeland and unconditionally supports a 60% reduction in carbon emissions. D) A recent poll showed that a party whose most prominent candidate supports preemptive war and minimal carbon restrictions is quite popular. E) Twenty years ago, a pacifist party successfully and popularly ruled the country. Answer: C 45. After studying a random sample of 1024 individuals who had smoked daily for at least three years and comparing the results of this study with the results of a study of 1024 individuals who had never smoked, a group of researchers concluded that habitual smoking causes increased difficulties in concentrating. Which of the following, if true, most severely weakens the researchers' conclusion? A) The addiction to smoking and the cravings this addiction engenders is often on the mind of habitual smokers. B) Some non-smokers with attention deficit disorder (ADD), which causes an inability to concentrate, display even less ability to concentrate than some smokers. C) A separate research study found that smokers and non-smokers exhibited statistically significant differences in their incarceration rates. D) After developing a severe addiction to smoking for fifteen years, the ability of many individuals to concentrate is decreased. E) A separate research study found that individuals with preexisting attention and concentration disorders exhibited significantly higher rates of trying cigarettes and subsequently becoming addicted to smoking. Answer: E 46. Political Commentator: During the previous presidential administration, members of congress approved large tax cuts and yet the economy today stands in shambles. During the current economic crisis, those who espouse large tax cuts as an economic stimulus should consider the failure of tax cuts during the past eight years to prevent the current economic recession as conclusive evidence that tax cuts will not help the country escape from its current economic troubles. Which of the following, if true, most weakens the argument above? A) The large tax cuts of the past administration targeted the capital gains on the financial investments of the ultra-rich while proponents of tax cuts today suggest cutting payroll taxes for both employers and employees. B) Economists from across the spectrum now agree that tax cuts helped stem the country's most severe recession in history, which occurred about 70 years ago prior to industrialization in this country. C) Economists from across the spectrum predict that if Congress fails to pass the tax-cut legislation, which also includes government spending and much more, it could well be 15 years before the economy escapes the current recession. D) Economists from across the spectrum agree that these tax cuts will add less to the fiscal deficit than the tax cuts of the previous administration. E) During the most recent political campaign cycle, which featured many deceptive political attacks, the governor of a prosperous state attacked those who opposed tax cuts by citing his own state's ostensible prosperity. Answer: A 47. A recent article in one of the nation's leading newspapers noted that despite the government's warning about peanut butter likely being contaminated by salmonella and the government's subsequent recall of a limited amount of peanut butter, 90% of grocery store shoppers surveyed said that they did not plan to change their peanut butter purchasing habits. Nevertheless, roughly two months after the limited recall and one month after the leading newspaper published its article, the country's peanut butter manufacturers reported that same-store sales to grocery store shoppers fell 75% year-over-year. Which of the following, if true, best explains the apparent paradox above? A) The initial survey of shoppers failed to consider the effect of subsequent cuts in the price of peanut butter. B) Fearing additional instances of contamination and subsequent lawsuits, many retailers that sold peanut butter removed the product voluntarily from their shelves. C) A report similar to the report that appeared in the leading newspaper appeared in one of the nation's tabloid magazines on the same day. D) Days before the newspaper conducted its survey, a widely-respected bacterial research specialist published an op-ed article in a major newspaper arguing that the threat from salmonella-infected peanut butter was smaller than the government would later contend. E) A study published after the government recall of some peanut butter stated that individuals intended to change the type of jelly and bread they purchased. Answer: B 48. An advertisement for E-News, a subscription-only online newspaper with no ads accompanying its content, argued that individuals should subscribe to E-News so as to eliminate wasted time that results from seeing and ignoring advertisements while attempting to read newspapers that feature ads. Which of the following, if true, most severely weakens E-News' argument? A) Individuals who currently read only print newspapers are much less likely to enjoy a subscription news website without first learning about reading online news through a free news website. B) Free ad-sponsored news websites and blogs offer more content than E-News. C) E-News partners with other e-content providers, many of which always show numerous ads alongside their content. D) For regions of the country that lack adequate internet coverage, switching to an online subscription website is not logical. E) E-News recently announced it would sell some of its content to ad-supported websites and print newspapers. Answer: C 49. In an attempt to abate the pernicious decline in MicroChip's revenue brought about by shrinking demand that is accompanying an economic recession, MicroChip is offering customers a 50% discount for the next three months on all purchases fully paid for within 15 days. Which of the following assumptions most underlies the chip maker's offer of a discount? A) MicroChip expects this discount to help the company retain existing customers and gain new ones, enabling the firm to survive in the long-term. B) There are no other competing chip companies with prices lower than the reduced price. C) The government will provide massive technology tax credits to businesses, spurring them to purchase chips and other related products. D) The government will not pursue MircoChip if in fact its behavior in offering a deep discount amounts to a violation of predatory pricing laws. E) The decrease in revenue brought about by the reduction in price will be smaller than the anticipated increase in revenue brought about by the increase in demand (spurred by the reduction in price). Answer: E 50. Based upon the results of a recent study, the net value of assets held by young adults or for the benefit of young adults exceeds the net value of assets held by middle-age working professionals with children. The common notion that young adults or so-called "twenty somethings" are bigger spenders and smaller savers than middle-age adults is, therefore, false. The argument is primarily flawed for which of the following reasons? A) The argument does not properly consider the impact of the debt financing of assets. B) The argument never discusses the effects of filing for bankruptcy and twenty somethings' proclivity for deficit spending leading to bankruptcy. C) The argument never discusses the role that the country's tax code, which encourages financial investment on the part of twenty somethings, plays. D) The argument does not specify the exact amounts of saving and spending on the part of each age group. E) The argument never considers that the study compares assets held both by or for the benefit of young adults with assets held by working adults. Answer: E
C. Sentence Correction You are given a sentence having an underlined portion and five answer choices.
  • If the sentence sounds right as it is, option 1 is the appropriate answer
  • You have to be familiar with the rules of English grammar and pick the appropriate answer which appears the best in compliance with standard written English.
  • Once done, read it over again to check if the sentence development looks correct and if the first meaning of the announcement has been held.
1. Trying to keep her balance on the icy surface, the last competitor's ski-tip caught the pole and somersaulted into the soft snow. A. the last competitor's ski-tip caught the pole and somersaulted into the soft snow. B. the ski-tip of the last competitor caught the pole and somersaulted in the soft snow. C. the last competitor caught the pole with the tip of her ski, and somersaulted into the soft snow. D. the last competitor caught the pole with her ski-tip, which made her somersault into the soft snow. E. the last competitor somersaulted into the soft snow when the tip of her ski was caught by the pole. Answer: C 2. The temperature dropped suddenly last night, which will mean that the shoots emerging from the soil will be killed by the frost. A. which will mean that the shoots emerging from the soil will be killed by the frost. B. which will mean that the frost will kill the shoots emerging from the soil. C. and this will mean that the shoots emerging from the soil will be killed by the frost. D. and the resulting frost will kill the shoots that are emerging from the soil. E. and as a result, the shoots will be killed by the frost, emerging from the soil. Answer: D 3. The impostor eluded detection for so long because she conducted herself as though she were a licensed practitioner. A. as though she were a licensed practitioner. B. as though she was a licensed practitioner. C. like she was a licensed practitioner. D. like as if she was a licensed practitioner. E. as if she was a practitioner with a license. Answer: A 4. Being abandoned by our friends is the cause of great sorrow for us. A. Being abandoned by our friends is the cause of great sorrow for us. B. Our being abandoned by our friends is the cause of great sorrow. C. Being abandoned by our friends, we feel great sorrow. D. Abandoned by our friends, sorrow is the result. E. We feel great sorrow when our friends abandon us. Answer: E 5. Among the many reasons for his defeat in the election was his arrogant assumption that his constituents were incapable of understanding economic conditions, and his unwarranted attack on his chief opponent. A. was his arrogant assumption that his constituents were incapable of understanding economic conditions B. were his arrogant assumption that his constituents were incapable of understanding economic conditions C. were his arrogant assumptions that his constituents were incapable of understanding economical conditions D. were his arrogant assumption that his constituents would be incapable of understanding economics E. was the arrogant assumption that his constituents was incapable of understanding economic conditions Answer: B 6. More and more holidaymakers are choosing to fly to remote islands in search of the perfect beach; seeking sand, sun and palm trees, rather than centers of entertainment. A. ; seeking sand, sun and palm trees, rather than centers of entertainment. B. ; seeking sad, sun, palm trees and not entertainment. C. , with sand, sun, palm trees and no entertainment. D. , they seek sand, sun and palm trees, rather than entertainment centers. E. ; they seek sand, sun and palm trees, rather than centers of entertainment. Answer: E 7. The government requires that these forms should be submitted before the end of the financial year. A. that these forms should be submitted B. that these forms be submitted C. for these forms to be submitted D. these forms submission E. these forms should be submitted Answer: B 8. After arduous months of fighting, the sight of the white flag being raised generated as much relief on the victor's side than it did on the vanquished. A. as much relief on the victor's side than it did on the vanquished. B. as much relief among the victors as among the vanquished. C. as much relief on the victor's side as it did on the vanquished's. D. relief both on the victor's side as well as on the vanquished's. E. relief both for the victor and the vanquished side. Answer: B 9. The best way to encourage innovative thinking is not to promise financial rewards for ideas, but to ensure that the person making the suggestion receives recognition for his contribution. A. but to ensure that the person making the suggestion receives recognition for his contribution. B. but to ensure that the person who makes the suggestion will be receiving recognition for his contribution. C. but rather by ensuring that the person making the suggestion receives recognition for his contribution. D. but rather ensure that suggestion-maker receives recognition for his contribution. E. but instead make sure that the suggestion-maker will receive recognition. Answer: A 10. It ought to be her with whom you share your secrets, not me. A. her with whom you share your secrets, not me B. her with whom you share your secrets, not I. C. she with whom you share your secrets, not me. D. she with whom you share your secrets, not I. E. her with who you share your secrets, not me. Answer: A 11. A conjunction is used to connect words and sentences together. A. words and sentences together. B. words or sentences together. C. words and sentences. D. words or sentences. E. words to sentences. Answer: D 12. Antony, coming alongside Cleopatra's ship, climbed aboard without seeing or being seen by her. A. climbed aboard without seeing or being seen by her. B. climbed aboard without seeing Cleopatra or being seen by her. C. climbs aboard without Cleopatra seeing him. D. boarded without being seen by her. E. boarded without seeing Cleopatra. Answer: B 13. Bombast is when high sounding words for effect, not suitability, are used. A. is when high sounding words for effect, not suitability, are used. B. is the use of high-sounding words for effect rather than for suitability. C. is where high-sounding words are used for effect not suitability. D. is the using of high-sounding words for effect only. E. is when you use high-sounding words for effect rather than for suitability. Answer: B 14. I would like to thank whoever it was that wrote that piece of music: it has given me so much pleasure. A. I would like to thank whoever it was that wrote that piece of music: B. I would like to thank whomever it was that has written that piece of music: C. I would like to thank whomever it might be that wrote that piece of music: D. Whoever it was that wrote that piece of music, I would like to thank because E. I would like to thank whoever it was that wrote that piece of music, Answer: A 15. Your taking a loan to buy a car annoyed Father. A. Your taking a loan to buy a car annoyed Father. B. Your taking a loan to buy a car aggravated Father. C. You taking a loan to buy a car irritated Father. D. You taking a loan to buy a car annoyed Father. E. Father was annoyed by you taking a loan to buy a car. Answer: A 16. In the initial stages of learning a new language we learn more through listening and attempting to copy speech patterns and not through reading grammar books. A. and attempting to copy speech patterns and not through reading grammar books. B. and attempting to copy speech patterns than through reading grammar books. C. and attempts to copy speech patterns than through reading grammar books. D. and attempts at copying speech patterns than through reading grammar books. E. and attempts at copying speech patterns and not grammar books. Answer: B 17. Between you and I, I doubt that he will come. A. and I, I doubt that he will come. B. and I, I doubt that he would come. C. and me, I doubt that he will come. D. and me, I doubt that he would come. E. and me, I doubt whether he will be coming. Answer: C 18. Work that is not finished is not work at all, it is merely a botch, a failure. A. all, it is merely a botch, a failure. B. all, it is a botch merely, and a failure. C. all; it is merely a botch, a failure. D. all; the work merely is a botch and a failure. E. all; the work being merely a botch, a failure. Answer: C 19. Pollution and degradation of the environment is, according to the commission's report, a matter of universal concern. A. is, according to the commission's report, a matter of universal concern. B. is, according to the report by the commission, a matter of universal concern. C. is, according to the report of the commission, matters of universal concern. D. are, according to the commission's report, universally a matter of concern. E. are, according to the commission's report, a matter of universal concern. Answer: E 20. Denim jeans were originally worn not so much as a fashion statement as for their being practical work clothes. A. Denim jeans were originally worn not so much as a fashion statement as for their being practical work clothes. B. Denim jeans were originally worn not so much as a fashion statement but for their being practical work clothes. C. Denim jeans were originally worn not so much as a fashion statement but for being practical work clothes D. Denim jeans were originally worn not as a fashion statement as for them being practical work clothes. E. Denim jeans were originally worn not as a fashion statement but as for them being practical work clothes. Answer: A 21. It is highly desirable that you furnish evidence of your expenses before you submit your final accounts. A. It is highly desirable that you furnish evidence of your expenses B. It is highly desirable that you should furnish evidence of your expenses C. It is highly to be desired that you furnish evidences of your expenses D. You must furnish evidence of your expenses E. You should have to supply evidence of your expenses Answer: A 22. The population of tigers in the National Park is increasing steadily, and this is a source of encouragement to those who have worked so hard to fund the conservation effort. A. steadily, and this B. steadily: which C. steadily; this trend D. steadily, this increase E. steady, and this Answer: C 23. In the fine print at the end of the document lies the clauses that make us liable for any expenses that result from civil unrest. A. lies the clauses that make us liable for any expenses that B. lies the clauses that make us liable for any expenses which C. lies the clause that make us liable for any expenses that D. lie the clauses that makes us liable for any expenses which E. lie the clauses that make us liable for any expenses that Answer: E 24. The administration discussed whether the number of students studying European languages was likely to decline when the senior lecturer retired. A. whether the number of students studying European languages was likely B. whether the number of students studying European languages were likely C. if the students studying European languages were likely D. if the number of European language students were likely E. whether the number of students studying European languages was liable Answer: A 25. If the gardener would sow the seeds in the greenhouse rather than the garden, he might get a better display of flowers. A. If the gardener would sow the seeds in the greenhouse rather than the garden B. If the gardener sowed the seeds in the greenhouse rather than the garden C. If the gardener would sow the seeds in the greenhouse rather than in the garden D. If the gardener were to sow the seeds in the greenhouse rather than in the garden E. If the gardener would sow the seeds in the greenhouse instead of the garden Answer: D 26. On Discovery channel last night they showed an informative program about new innovations in medical imaging, which you would have found interesting. A. they showed an informative program about new innovations in medical imaging, which you would have found interesting. B. they showed an informative program about innovations in medical imaging, which you would have found interesting. C. they showed an informative program about innovations in medical imaging, that you would have found interesting. D. there was an informative program about new innovations in medical imaging, that you would have found interesting. E. there was an informative program about innovations in medical imaging, a program you would have found interesting. Answer: E 27. After working for two hours, the essay started to take shape, and he began to hope that he might finish before the deadline. A. After working for two hours, the essay started to take shape, and he began to hope B. When the essay started to take shape after he had worked on it for two hours, he began to hope C. When the essay started to take shape after he had worked on it for two hours, he begun to hope D. When the essay started to take shape after working on it for two hours, he began to hope E. After working for two hours, the student's essay started to take shape, and he began to hope Answer: B 28. The young man was surprised to find that his experience as a tutor had been used as the basis for the protagonist in a short story written by a former girlfriend. A. that his experience as a tutor had been used as the basis for the protagonist in B. his experience as a tutor having been used as the protagonist in C. his experience as a tutor had been used as the basis for the protagonist's in D. his experience as a tutor being used as the basis for the protagonist of E. that his experience as a tutor had been used as the basis for events in the life of the protagonist in Answer: E 29. Similar to the Incan Temple at Winay Wayna, the Maya culture constructed pyramids to use as observatories, religious shrines, and political centers, like those at Tikal, located in the north of Guatemala. A. Similar to the Incan Temple at Winay Wayna, the Maya culture constructed pyramids to use as observatories, religious shrines, and political centers, like those B. Just like the Incan Temple at Winay Wayna, the constructed ones in Maya culture were used as observatories, religious shrines, and political centers C. As the Incan Temple at Winay Wayna was constructed for use as an observatory, religious shrine, and political center, so the Maya D. Like the Inca, who constructed the Temple at Winay Wayna, the Maya used pyramids as observatories, religious shrines, and political centers, such as those E. So with the Inca, who constructed the Temple at Winay Wayna, the Maya constructed pyramids for use as observatories, religious shrines, and political centers, like those Answer: D 30. The mole is a nocturnal insectivorous mammal regarded as pests by gardeners because of their burrowing activity spoiling lawns and gardens. A. regarded as pests by gardeners because of their burrowing activity spoiling B. regarded to be pests by gardeners because of their burrowing activity's spoiling C. regarded as a pest by gardeners because of burrowing activity spoiling D. considered as a pest by gardeners because of its burrowing activity spoiling E. regarded as a pest by gardeners because its burrowing activity spoils lawns and gardens. Answer: E 31. With his sub-four minute mile Bannister broke a psychological barrier, inspiring thousands of others to attempt overcoming seemingly insurmountable hurdles. A. inspiring thousands of others to attempt overcoming B. inspiring thousands of others to attempt to overcome C. inspiring thousands of others to overcome D. and inspired thousands of others to attempt to overcome E. and inspired thousands of others to attempt overcoming Answer: D 32. Ricks has written extensively on not only major figures in English poetry like Milton and Housman, but also on the lyrics of Bob Dylan. A. on not only major figures in English poetry like Milton, but also on B. not only on the poetry of such major figures as Milton and Housman, but also on C. not only on major figures in English poetry like Milton and Housman, but also on D. on major figures in English poetry like Milton and Housman, as well as E. on major figures in English poetry such as Milton and Housman, but also on Answer: B 33. An analysis of sixteenth century probate inventories in the major English towns show that even some artisans and yeomen owned silver spoons, cups or salt cellars. A. show that even some artisans and yeomen owned silver spoons, cups or B. show that some artisans and yeomen even owned silver spoons, cups or C. show that even some artisans and yeomen owned silver spoons, cups and D. shows that some artisans and yeomen owned even silver spoons, cups and E. shows that even some artisans and yeomen owned silver spoons, cups or Answer: E 34. Stress-induced amnesia is a rare phenomenon; it strikes the patient apparently without warning and the memory loss can be as complete as that induced by physical trauma. A. as complete as that induced by physical trauma B. as complete as is induced by physical traumas C. as least as complete as physical trauma induces D. at least as complete as physical trauma E. just as complete as those induced by physical trauma Answer: A 35. The publishers, unwilling to shoulder the entire risk, insisted that the author should pay half the cost of the initial print run of his controversial new book. A. The publishers, unwilling to shoulder the entire risk, insisted that the author should pay half the cost of the initial print run of his controversial new book. B. The publishers, unwilling to shoulder the entire risk, insisted that the author should be paying half the cost of the initial print run of the author’s controversial new book. C. The publishers, unwilling to shoulder the entire risk, insisted that the author pay half the cost of the initial print run of his controversial new book. D. Unwilling to shoulder the entire risk, the publishers insisted the author should pay half the cost of the initial print run of his controversial new book. E. Unwilling to shoulder the entire risk, the author was required by the publisher to pay half the cost of the initial print run of his controversial new book. Answer: C 36. It is probable that the prototype cellular motor might be ready for testing around the end of next year. A. might be ready for testing around the end of next year B. may be ready for testing about the end of next year C. might be ready for testing toward next years end D. will be ready for testing toward the end of next year E. should be ready for testing toward the end of next year Answer: D 37. Making use of contemporary diaries and letters, Florey’s complexity is brilliantly revealed by Lax. A. Florey’s complexity is brilliantly revealed by Lax B. Florey is brilliantly shown in all his complexity by Lax C. Lax brilliantly reveals Florey’s complexity D. Lax brilliantly revealed the complexity of Florey E. Laxs study has brilliantly revealed the complexity of Florey Answer: C 38. One of the perennial problems of the tourist industry is that of fitting what people want to see into the time they have available to see it in. A. that of fitting what people want to see into the time they have available to see it in B. fitting what people want to see into the time they have available C. that of fitting what people want to see in the time which they have available D. fitting what people want to see in the time they have available for seeing E. the need to fit what people want to see into the available time for seeing Answer: B 39. If the engineering feats of that early decade remains impressive boring a mile-long tunnel through a solid mountain, and turning the eastward flow of two rivers westward the arduous conditions in which the engineers worked are no less remarkable. A. If the engineering feats of that early decade remains impressive boring B. If the engineering feats of that early decade remains impressive such as the boring of C. Although the engineering feats of that early decade remains impressive boring D. If the feats of engineering in that early decade remains impressive boring E. If the engineering feats of that early decade remain impressive boring Answer: E 40. The United Nations Human Development Index takes into account life expectancy, education, as well as income per person. A. into account life expectancy, education, as well as income per person B. life expectancy, education, as well as income per person into account C. into account life expectancy and education, as well as income per person D. into account life expectancy, and education, and income per person E. life expectancy, education, and income per person in its account Answer: C 41. The psychologist reported that the most common problems undergraduates experience stem from a lack of self-esteem, difficulty in adjusting to university life and loneliness. A. the most common problems undergraduates experience stem from a lack of self-esteem, difficulty in adjusting to university life and loneliness B. the most common problems undergraduates experienced stemmed from a lack of self-esteem, difficulty in adjusting to university life and loneliness C. the most common problems undergraduates experience stem from loneliness, a lack of self-esteem, and difficulty in adjusting to university life D. the most common problem undergraduates have experienced stems from a lack of self-esteem, loneliness, and difficulty in adjusting to university life E. the most common problems undergraduates had experienced stem from difficulty in adjusting to university life, lack of self-esteem, and loneliness Answer: C 42. Inertia-gravity waves cause characteristic stripy patterns in the clouds in the lower atmosphere but they are disregarded by conventional weather forecasts because they are thought to be too small to interact with larger systems such as warm and cold fronts. A. they are disregarded by conventional weather forecasts because they are thought to be too small B. they are disregarded by conventional weather forecasts because these waves are thought to be too small C. conventional weather forecasts disregard them because they think they are too small D. conventional weather forecasts disregard these waves because they are thought to be too small E. conventional weather forecasts think them too small Answer: D 43. In archeological terms the university was a latecomer to the town, which was already centuries old by the time we first hear of the establishment of a community of scholars and teachers in the late 12th Century. A. which was already centuries old by the time we first hear of the establishment of B. already centuries old by the time we first hear of its establishment of C. which was centuries old already when we first hear of the establishment of D. that was already centuries old by the time we first are hearing of the establishing of E. that was already centuries old by the time we first hear that they had established Answer: A 44. The recent photographs of the giant squid are remarkable because they show these enormous living creatures as moving around in their natural environment, whereas previous pictures have been of only dead animals. A. because they show these living creatures as moving around in their B. in that they show this most enormous of living creatures moving around in its C. in that they show this enormous living creature moving around in its D. because these enormous living creatures are shown to be moving around in their E. because they show this enormous living creature moving around in a Answer: C 45. It is usual for scientists and social scientists to abandon their theories only if another more attractive theory comes along and not when they have been proved incorrect. A. their theories only if another more attractive theory comes along and not when they have been proved incorrect B. a theory not because it has been proved incorrect, but because another more attractive theory comes along C. their theories not when they have been proved incorrect, but because other more attractive theories come along D. a theory only when other more attractive ones come along, rather than when they are proved wrong E. their theories not if they have been proved wrong, but if another more attractive theory comes along Answer: B 46. Once a hurricane is identified, it is given a name from a list drawn up by the United States Weather Service, a list that is reused after a few years, but with the names of the worst hurricanes omitted. A. but with the names of the worst hurricanes omitted B. omitting the names of the worst hurricanes C. the names of the worst hurricanes being omitted D. after they have omitted the names of the worst hurricanes E. after omitting the worst hurricane names Answer: A 47. Dunbar argues that gossip is important in human societies in the maintenance of social cohesion, just as social grooming does for other primates. A. gossip is important in human societies in the maintenance of social cohesion, just as social grooming does for other primates B. gossip is important in the maintenance of social cohesion in human society, just as social grooming does for other primates C. in human societies gossip is important in social cohesion like social grooming for other primate societies D. similar to social grooming in primates, human gossip is important in maintaining social cohesion E. gossip is important in human societies in the maintenance of social cohesion, just as social grooming is in other primate groups Answer: E 48. Chinas economy continues to flourish this year: industrial production grew, inflation has eased, and the trade surplus swelled. A. grew, inflation has eased, and the trade surplus swelled B. has grown, inflation has eased, and the trade surplus swelled C. has grown, inflation eased, and the trade surplus has swelled D. has grown, inflation eased, and the trade surplus is swelling E. is growing, inflation easing and the trade surplus swelling Answer: E 49. The Bengal school attempted to create an authentic style of Indian painting based on the study of the art of ancient India, Indian medieval miniature traditions, as well as European and Japanese art. A. Indian medieval miniature traditions, as well as European and Japanese art B. and Indian medieval miniature traditions, as well as European and Japanese art C. Indian medieval miniature traditions, and European and Japanese art as well D. and Indian medieval miniature traditions, and European as well as Japanese art E. Indian medieval miniature traditions, European, and Japanese art Answer: B 50. Often said to be the most perfectly cut of all large diamonds, the Jubilees facets are so exact that it can be balanced on its culet, which is less than two millimeters across. A. the Jubilees facets are so exact that it can be balanced on its culet, which is less B. the Jubilees facets are so exact that the diamond can be balanced on its culet, less C. the Jubilee has facets so exact that it can be balanced on its culet, which is less D. the Jubilee has facets which are so exact that it can balance on its culet, which is less E. the Jubilees facets are so exact that it can be balanced on its culet, which is lesser Answer: C